Você está na página 1de 73

Homi Bhabha Centre

for Science & Education


(HBCSE)

Indian National Astronomy Olympiad (INAO)


Year 2010 to 2015 Stage-2 Test Papers
with Solutions
INDIAN NATIONAL ASTRONOMY OLYMPIAD
(INAO) 2010-2015

CONTENTS

TOPICS PAGE NO.

1. INAO Test Paper 2010 01 - 06

2. Answers & Solutions 07 - 12

3. INAO Test Paper 2011 13 - 18

4. Answers & Solutions 19 - 26

5. INAO Test Paper 2012 27 - 33

6. Answers & Solutions 34 - 39

7. INAO Test Paper 2013 40 - 42

8. Answers & Solutions 43 - 47

9. INAO Test Paper 2014 48 - 52

10. Answers & Solutions 53 - 57

11. INAO Test Paper 2015 58 - 63

12. Answers & Solutions 64 - 70


16RDLP

Copyright reserved.
All rights reserved. Any photocopying, publishing or reproduction of full or any part of this study
material is strictly prohibited. This material belongs to enrolled student of RESONANCE only. Any
sale/resale of this material is punishable under law, subject to Kota Jurisdiction only.
1

INDIAN NATIONAL ASTRONOMY OLYMPIAD


(INAO)-2010
INAO 2010 Date: 30th January 2010
Duration: Three Hours Maximum Marks: 100

Please Note:
Please write your roll number on top of this page in the space provided.
Before starting, please ensure that you have received a copy of the
question paper containing total 3 pages (6 sides).
In Section A, there are 10 multiple choice questions with 4 alternatives
out of which only 1 is correct. You get 3 marks for each correct answer
and -1 mark for each wrong answer.
In Section B, there are 4 multiple choice questions with 4 alternatives
each, out of which any number of alternatives may be correct. You get
5 marks for each correct answer. No marks are deducted for any wrong
answers. You will get credit for the question if and only if you mark all
correct choices and no wrong choices. There is no partial credit.
For both these sections, you have to indicate the answers on the page
2 of the answersheet by putting a in the appropriate box against the
relevant question number, like this:

Marking a cross () means affirmative response (selecting the particular


choice).
Do not use ticks or any other signs to mark the correct answers.
In Section C, there are 5 analytical questions totalling 50 marks.
Blank spaces are provided in the question paper for the rough work. No
rough work should be done on the answer-sheet.
No computational aides like calculators, log tables, slide rule etc. are
allowed.
The answer-sheet must be returned to the invigilator. You can take this
question booklet back with you.
2

Section 1 : Multiple Choce Questions


Part A : (10Q 3 marks each)
1 As seen from the Earth, the stars appear to twinkle, while the planets do
not because,
(A) Light coming from the stars gets absorbed by interstellar dust.
(B) The stars are self luminous while the planets merely reflect the
light.
(C) Angular sizes of the stars are much smaller than the planets.
(D) All the above.

A wooden cube of length 5 units is painted on all faces and then cut
in 85 smaller cubes of varying sizes. The sides of each of the smaller
cubes are some integer number of units. All bigger cubes are carved
out of the corners of the original cube. Answer the following three
questions:

2. How many cubes have no side painted?


(A) 15 (B) 17 (C) 20 (D) 27
3. How many cubes have exactly 2 sides painted?
(A) 24 (B) 27 (C) 30 (D) 36
4. If one was to cut the original cube (of size 5 units each side) with a
condition that all corners would be occupied by the cubes bigger than
1 unit size, what will be the smallest total of cube pieces possible?
(A) 69 (B) 64 (C) 62 (D) 50
5. It is easier to balance on a faster bicycle moving along a straight line
than a slower one because of
(A) conservation of linear momentum
(B) conservation of angular momentum
(C) conservation of linear momentum and angular momentum
(D) conservation of energy
6. On a nice sunny day in Chennai, Akshay saw a supersonic fighter plane
flying parallel to the ground in the sky. As a student of Aeronautical
Engineering, he knew off -hand the speed of the plane to be 1.25 Mach
(i.e. 1.25 times the speed of sound in air). He could hear its sonic boom,
12 seconds after the plane flew directly overhead. What is the altitude of
the plane'?
(A) 3.7 Km (B) 4.0 Km (C) 5.3 Kin (D) 6.6 Km

7. If you hold a magnifying glass of focal length 10 cm in. the sunlight and
place a piece of paper at its focus, you can burn a hole in the paper. What
could be the size of this hole?
(A) 10 mm (B) 5 mzn (C) 0.5 mm (D) 0.1 mm
3
8. During an earthquake, an earthquake monitoring Centre observed that
transverse waves traveling with speed 4.5 km/s arrived at the centre
3 minutes after the longitudinal waves traveling at 8.2 km/s. Deduce the
approximate distance to the epicenter.
(A) 60 km (B) 220 km (C) 660 km (D) 1800 km

9. A certain person nicknamed "Enthu", encountered an automatic staircase


(i.e. escalator) at a shopping complex, which was moving upward at a
constant; rate. Just for the fun of it he decided to walk up this escalator at
the rate of one step a second. Twenty steps brings him to the top. Next
day he goes up at two steps a second and reaching the top in 32 steps.
How many steps are there in the escalator?
(A) 40 (B) 60 (C) 64 (D) 80

10. Two stars of masses M and 3M respectively are going around each
other, in near circular orbits, with period T. The separation between

kGMT 2
them is given by D = 3 The value of k is,
2

(A) 0.5 (B) 1 (C) 1.5 (D) 3

Section B : ( 4 question 5 marks each)


11. If we throw a ball in a shallow water tank, propagation velocity of ripples
on surface of the water will depend on
(A) surface tension of the water
(B) depth of the water tank
(C) density of the water
(D) height from which the ball was dropped

12. Amit decided to experiment with cannon ball by making it hollow and
filling water inside it. He then punched few holes in it. After the ball was
fired horizontally, he was expecting to see water jets coming out from
some of the holes. Which of the following locations of holes will allow
water jet to come out?
(A) Front, bottom and sides (B) Back, top and sides
(C) back and bottom (D) None of the holes
4
13. In the progress of Astronomy over the ages, we find several instances of
startling new observations changing our ideas about the Universe and
lead to new theories. Listed below are some milestones in the history of
Astronomy and observations which necessitated them. Pick the correct
statement(s).
(A) Ptolemy was aware of retrograde motion of planets when he gave
his model of the solar system.
(B) Newton was aware of Kepler's Laws when he gave his Law of
Gravitation.
(C) Einstein felt need to modify Newton's theory of Gravitation to explain
the expansion of the Universe.
(D) Existence of Cosmic Microwave Background Radiation (CMBR) led
to the creation of the Big Bang Theory.

14. Two simple pendula are hung close to each other on a thin, rigid support
and are allowed to oscillate independently in planes parallel to each
other. They have periods 3 seconds and 7 seconds respectively with
same amplitudes and arc initially released from the opposite extreme
positions of each other. At which of the following times the threads of the
two pendula will be coplanar again?
(A) 1.05 seconds (B) 7.88 seconds
(C) 10.50 seconds (D) 23.10 seconds

Section C: Analytical Questions


An alien civilisation on a star far far away came to know about the
Astronomy Olympiad Examination and wanted to test smartness of the
students. They sent following two coded secret messages. Decode
them.
(a) (5 marks) First is a pictorial message in black and white colour.
They sent it on radio waves in the form of ones and zeros. Find out
what. the picture says.
00000 00000 00000 00000 00000 00000 11111 01100 00100 00100
00011 10000 01000 10100 01000 10100 01101 10000 10001
00100 10001 11000 10001 00001 00010 00101 00100 01001
10110 01111 10100 00110 10000 01001 11000 00000 00000 00000
00000 00000 000
(b) (5 marks) Surprisingly. the aliens are also proficient in English and
the following coded message is actually a sentence in English
language. In the answer sheet, write down the coded sentence and
also the true meaning of each alphabet in the code0.
"Up tpmwf uif qsfwjpvt tvcrvftujpo uijol pg uif nfttbhf tfou cz uif bsfdjcp
ufmftdpgf up bmjfot."
5
(8 marks) In the following table, the first column gives various optical
phenomena / instruments and the top row gives various optical effects
which may help in explaining them. In the answers sheet, tick the correct
effect(s) involved in each phenomena in appropriate rows.
Appreciable Internal
Reflection Refraction Scattering
Dispersion Reflection
Blue Sky
Mirage
Rainbow
Smooth Convex Mirror
Thick Concave Lens

Study the following image of the night sky (bigger version is printed in
your answer-sheet) for mid-night on a certain day at a certain place and
answer the questions below it. All answers must be marked / written on
the answer-sheet.

(a) (2 marks) Mark all the four directions on the map.


(b) (1 mark) Is this place in northern part of India or the southern part?
(c) (3 marks) Mark and name 2 constellations each from the following
two lists:
0.5 marks each; any 2: Orion, Ursa Major, Taurus, Leo, Cygnus,
Scorpio o
1 mark each; any 2: Hydra, Corvus, Aquarius, Cancer, Canis Major,
Aries
(d) (2 marks) Sketch rough band of ecliptic i.e. apparent path of the
Sun, the Moon and all the planets in the sky.
(e) (2 marks) In which month the sky will appear like this at this time?
Give reason in one line.
6
. Mayank visited a place located at latitude and longitude 82.5E on
21st June. Ile observed that at local noon, shadow of a one meter
stick standing vertically on the ground was 26.8 cm long due south.
(a) (5 marks) Find latitude of the place.
(b) (5 marks) Find the day on which the shadow of this stick at the
local noon will be longest and find length and direction of that
shadow.
Note: sin1 (0.268) = 15.5, cos1 (0.268) = 74.5, tan1(0.268) = 15.0,
1
tan(2) and 3 = 1.73
30
. Sketch approximate graphs for the following situations:
(a) (4 marks) See the figure below. A tank of water (height of water
column b) is kept on a electronic weighing scale. A metal cube
(side a and density p) is hung from a spring balance and the
spring balance is slowly lowered into the tank till the cube
reaches the bottom of the tank. The distance of separation
between the bottom of the tank and bottom of the cube is denoted
by h with initial value h0.

spring
balance

a
h
Water
tank
b

00.00
Weighing scale

Sketch the graph of reading on the spring balance as a function of h.


(b) (3 marks) For the situation above, sketch the graph of reading
on the electronic scale as a function of h.
(c) (2 marks) For the situation above, sketch the graph of sum of
the reading on the electronic scale and the reading on the spring
balance as a function of h.
(d) (3 marks) For a typical primary mirror used in Newtonian
telescope, sketch a graph of object distance, a versus image
distance, . All physically measurable lengths should be taken
as positive.
In all cases, mark the significant points on the graph and give
their coordinates.
7
INDIAN NATIONAL ASTRONOMY OLYMPIAD (INAO)-2010
ANSWERS & SOLUTIONS

Section 1 : Multiple Choce Questions


1 (C)
2. (B)
If you cut the bigger cube in smaller cubes with all side of 1 unit, there
will be 125 total cubes.
Each cube of size 2 will fuse 8 such smaller cubes together. i.e. the total
number of cubes will reduce by 7.
Similarly, each cube of size 3 and 4 will reduce total number of cubes by
26 and 63 respectively.
Now, we have 85 total cubes, which is 40 less than 125. This means
there is 1 cube of size 3 and 2 cubes of size 2 at three of the corners.
Rest all are size 1.
If there were 125 smaller cubes of size 1, one can say the volume which
is not touched by paint on any face is equivalent to a cube of size 3 i.e.
27 smaller cubes. However, the the bigger cubes carved out of corners
include some of these inner cubes as well. 8 such inner smaller cubes
are included in the size 3 cube and 1 each is included in the 2 size
2 cubes.
Thus, number of unpainted cubes = 27 8 1 1 = 17

3. (A)
To have exactly 2 sides painted, the smaller cube should be along any of
the edges but not at the corner. There are total 12 edges to the original
cube. For each edge, there will be 3 cubes satisfying the condition if all
cubes were of size 1. How ever, there is a size 3 cube. which includes
2 such smaller cubes each along 3 of the edges.
Similarly, the 2 size 2 cubes will each include 1 such smaller cube each
along 3 of the edges.
Thus, total number of cubes =12 3 2 3 1 3 1 3 24

4. (D)
There are two possibilities which will satisfy this condition. Either all
corners will have cubes of size 2 or else there will be 1 cube of size
3 and rest all corners will have cubes of size 2.
There are total 8 corners.
In first case total number of cubes will be = 125 - 8 7 = 69
In second case, total number of cubes will be = 125 7 7 26 = 50
8
5. (B)

6. (D)
s
Half angle of the cone of the sonic boom is given by sin = P , where
p, is velocity of the plane and sis the speed of the sound in air.
s
sin = = 0.8
1.25 s
cos = 0.6
sin h 0 .8 h
tan = cos = p t ; =
0 .6 1.25 330 12

4 15 330
h = = 6600 mt. = 6.6 km
3

7. (C)
If the magnification is given by M, Object distance, Object size, Image
Distance, Image size by u, O, & I respectively,
I I f
M = = ;
7 108

u O 500

0.1 7 108 7
10 3 mt. 0.5 mm
500 3 108 15

8. (D)
8.2 t= 4.5 x (t + 180)
3.7t = 810
810
D = 8.2t = 8.2 = 2.22 810
3.7
D 1800 km

9. (D)
Let the escalator rise with the speed of `n' steps per second.
On first clay, Enthu reaches the top after 20 manual steps i.e. in
20 seconds. On second day, Enthu reaches the top after
32 / 2 = 16 seconds. Thus, total number of steps,
20 (n + 1) = 16 (n + 2)
4n = 32 20 = 12
n =3
Steps = 20 x (3 + 1) = 80
9
10. (B)
D
The Centre of mass of the system will be located at distance from the
4
heavier mass. Centripetal force is provided by the mutual gravitation.
G M 3 M D 2
= (3M)
D2 4
GM D 4
= 4
D2 T2

GMT2
D3 =

GMT 2
D= 3

Section B : ( 4 question 5 marks each)
11. (ABC)
Note: The propagation velocity will vary as per the depth of the water,
provided the water is not too deep. However, the derivation for this effect
includes concepts beyond the level of participants. Thus, the accepted
solution for this question would be 'a' and 'c' must be selected, 'd' must
not be selected and 'b' would be acceptable both ways.

12. (D)
Horizontally, all particles (i.e. cannonball and water particles) are flying
with the same velocity. Hence water cannot come out of
front / back side holes. Vertically, all particles are experiencing free fall.
Thus, gravity cannot exert any additional pressure on water and neither
can water be slower than the cannonball. Hence no water can come out
from top or bottom.

13. (AB)
Ptolemy needed epicyclic model of the solar system as that was the
only way to fit observed motions of the planets including the retrograde
motion. Kepler's laws enabled Newton to deduce nature of the law of
gravitation. Einstein gave General theory of relativity in 1916 and assumed
the Universe to be non-expanding as per then norm. Hubble discovered
the expansion of the Universe in late 20's forcing Einstein to revisit his
own theory.
Existence of CMBR, was one of the key predictions of the big bang
theory. The theory was formulated before CMBR was discovered.
14. (AB)
10

Section C: Analytical Questions


In 1974, a pictographic message was sent to outer space by radio
waves using Arecibo telescope. The total number 'digit's in the message
was product of two prime numbers. It was argued that any intelligent
being can rearrange the grid in a rectangle with sides measuring these
two prime numbers and then use blank square for `0' and filled square
for '1' to reveal the picture.
In the present message same idea is used. Total number of digits is
203 which is a semiprime number; i.e. it can only be divided as 29 x 7. If
the grid is rearranged in this way, following pattern is seen:

Marking Scheme:
Realising total number of digits are 203, which is a semi-prime number.
(2 marks)
Attempt to draw either 29 7 or 7 29 grid. (1 mark)
picking the message "INAO" . (2 marks)

Part (b) is a substitution cipher.


The word "uif" appears thrice in the sentence. Obviously, it stands for
"the". Looking at the two words, the substitution rule is inferred as "every
letter is substituted by the next letter" to create the coded message.
Deciphering,
"To solve the previous subquestion think of the message sent by the
arecibo telescope to aliens." Marking Scheme:

Mapping the alphabets and deciphered Sentence. (5 marks)


Mapping the alphabets and Deciphered Sentence, but with small
syntactic errors. (4.5 marks)
Mapping the alphabets only. (4 marks)
Deciphered Sentence but Alphabet mapping is not given. (3 marks)
Only partial decipherment with more than 10 letters. (2 marks)
Only partial decipherment with 7 10 letters. (1.5 marks)
Only partial decipherment with "the" deciphered correctly. (1 mark)
11
Appreciable Internal
Reflection Refraction Scattering
Dispersion Reflection
Blue Sky
Mirage
Rainbow
Smooth Convex Mirror
Thick Concave Lens

Blue Sky (1 mark): One mark for scattering. -0.5 for each additional tick mark.
Mirage (2 marks): One each for internal reflection and refraction. 0 marks
if internal reflection is not ticked.
Rainbow (2 marks): One for internal reflection; 0.5 each for dispersion
and refraction. 0 marks if internal reflection is not ticked.
Mirror (1 mark): One mark for reflection. 0 if any additional / other tick marks.
Thick Lens (2 marks): One each for dispersion and refraction.
Any options not mentioned above, do not carry either positive or negative
credit.
(a) From Top clockwise: East, North, West, South. Note the position of
Ursa Minor Constellation (i.e. Pole star) to the right of the map.
Further, Taurus is at the bottom and Leo is at the top. (0.5 marks
per direction)
If the direction pointers are off by small angle (less than 10),
overall 0.5 marks.
(b) Ursa Minor is very close to Horizon. Hence the latitude of the
place is not very high. Thus. the place is in Southern India.
( 0.5 if wrong reason is mentioned.)
(c) Cygnus, Scorpio, Aquarius are not on the map. Rest are clearly visible.
(d) The band of ecliptic should pass through all the zodiac signs (i.e.
Aires to Virgo). Better precision than this is not expected. For every
zodiac sign significantly away from the ecliptic,
(- 0.5 marks (minimum zero).
(e) At mid-night, Cancer is nearly at Zenith. Hence the Sun is roughly
6 zodiac signs away i.e. in Capricorn. Thus, it will reach Vernal
Equinox (in Pisces) in roughly two months. Hence, current date is
about two months before 21st March. Thus, the current month is
January. 0.5 marks for getting month (December - February
accepted). Reason 1.5 marks.
. Shadow is 0.268 rn due South on the day of the Summer solstice. This
means the Sun is to the North of Zenith, i.e. the place is south of tropic of
cancer i.e. in southren India. (1 mark)
On that day, the Sun is directly above the tropic of cancer, i.e.
90- 23.5 = 66.5away from the North Celestial Pole. (1 mark)
If the altitude of the Sun at the local noon is 0,
1 1
tan Jun = = = tan(75)
0268 tan (15 )
= 75 66.5
= 8.5 (3 marks)
12
In part (b) For a northern hemisphere place, the longest shadow of the
stick will be cast on the Winter Solstice day. The Sun at local noon will be
47 further south than its position on the Summer Solstice clay. Hence
the Sun will be 180 75 47 = 58 high on the South of the Zenith.
(2 marks)
1 tan 30 tan 2
= = tan 32 = tan(30 + 2) =
tan 58 1 tan 30 tan 2
1 1

3 30 30 3 31.73 31.73
1 1 0.62 m
1 30 3 1 51.9 1 51
3 30

w w
3
a
3
a

. a3( 1) a
3

(0.0) (b a) b h0 h
(0.0) (b a) b h0 h

2f
w
f
3
a
f u

3
a ( 1)

(0.0) (b a) b h0 h

Partial marks given on basis of shape of the curve, neatness of figure,


proper nomencleture (axis titles etc.), proper markingof points and writing
their co-ordinates etc.
In (a), (b) and (c), it is acceptable to take mass of water as rn instead of
zero. No points cut.
The y-coordinates can be either in mass units or in force units
(multiplication by g). No points cut.
Density of water can be written as p,,, or its CCS values (i.e. 1) or its MKS
value (i.e. 1000). No points cut.
In. (d), the curve between = 0 and u = f may be plotted in both quadrant
1 and quadrant 2. No points cut.
13

Indian National Astronomy Olympiad (INAO) 2011


INAO 2011 DATE: 29TH JANUARY 2011
DURATION: THREE HOURS MAXIMUM MARKS: 100

Please Note:
Please write your roll number on top of this page in the space provided.
Before starting, please ensure that you have received a copy of the
question paper containing total 3 pages (6 sides).
In Section A, there are 10 multiple choice questions with 4 alternatives
out of which only 1 is correct. You get 3 marks for each correct answer
and -1 mark for each wrong answer.
In Section B, there are 4 multiple choice questions with 4 alternatives
each, out of which any number of alternatives may be correct. You get
5 marks for each correct answer. No marks are deducted for any wrong
answers. You will get credit for the question if and only if you mark all
correct choices and no wrong choices. There is no partial credit.
For both these sections, you have to indicate the answers on the page
2 of the answersheet by putting a in the appropriate box against the
relevant question number, like this:

Marking a cross () means affirmative response (selecting the particular


choice).
Do not use ticks or any other signs to mark the correct answers.
In Section C, there are 5 analytical questions totalling 50 marks.
Blank spaces are provided in the question paper for the rough work. No
rough work should be done on the answer-sheet.
No computational aides like calculators, log tables, slide rule etc. are
allowed.
The answer-sheet must be returned to the invigilator. You can take this
question booklet back with you.
14
SECTION 1: MULTIPLE CHOICE QUESTIONS

1. On one starry evening, Nidhi was trying to spot an artificial polar satellite
from her backyard. Typical altitude of any polar satellite is about 800km
above surface of the earth. What is the typical duration after sunset for
which Nidhi should try her luck?
(A) 63 min (B) 109 min (C) 127 min (D) 171. min

2. What will be the difference in potential energy (.6.U) of an object of mass


'M', if it is lifted from the ground to a height of 2R, where R is the radius of
the earth

2GM 2GM GM GM
(A) (B) (C) (D)
R 3R 2R 3R

3. Pole star appears stationary because


(A) Earth is not moving with respect to the pole star.
(B) Earth is on the axis of rotation of the pole star.
(C) Both Earth and the pole star have same velocity in the Milky Way
galaxy,
(D) None of the above

4. Consider a system of two converging lenses, one with focal length of


20cm and the other with focal length of 5cm, kept 50cm apart. An object
is kept at 40cm from the first lens. What can be said about the image
formed on the other side of the second lens?
(A) Erect and Real (B) Inverted and Real
(C) Erect and Virtual (D) Inverted and Virtual

5. Three rings of same dimensions, are dropped at the same time over
identical cylindrical magnets as shown below. The inner diameter of
each ring is greater than the diameter of the magnet.

Which of the following correctly describes the order in which the rings P,
Q and R reach the bottom of the respective magnets?
(A) They arrive in the order P, Q, R
(B) They arrive in the order P, R, Q
(C) Rings P and R arrive simultaneously, followed by Q.
(D) Rings Q and R arrive simultaneously, followed by P.
15

6. A charged particle with initial velocity V enters a region with a uniform

magnetic field B B i . If it starts moving along the positive X-axis in a
helical path such that the separation between successive loops is

constant, what can be inferred about V ?

(A) V Vj (B) V Vk (C) V V j V k (D) None of these
y z

7. If PQRS x 4 = SRQP, where P, Q, R and S are distinct non-zero digits.


what is value of R?
(A) 1 (B) 3 (C) 5 (D) 7

8. From the given P-V diagram, find out the total work done by the gas,
while going from state A to state C.

(A) W tot = W AC (B) W tot = W BC - W AB


(C) W tot = W AB + W BC (D) W tot = W AB W BC

9. Find out the equivalent resistance at AB from given circuit, if


R = 10

(A) 10 (B) 100 (C) 5 (D) 6


10. How many 3 digit prime numbers can be formed, using digits 5, 6 and
7 ? Repetition of digit is allowed.
(A) 20 (B) 4 (C) 7 (D) 6
16

Section B: (4 questions 5 marks each)


11. Four conducting plates A, B, C, D are arranged as shown in the figure.
Plates A and C are connected to the positive terminal of a DC source and
the Plates B and D are connected to the negative terminal of a DC
source. A proton is kep, at the centre of this assembly. If we disturb the
proton slightly from its equilibrium position, which of the following
statements will describe the path followed by the proton.

(A) If the proton is displaced slightly towards plate A, it will keep moving
towards plate A.
(B) If the proton is displaced slightly towards plate D, it will keep moving
towards plate D.
(C) If the proton is displaced slightly along diagonal of the assembly
between plates B and C, it will move directly towards plate B.
(D) If the proton is displaced slightly towards plate C, it will come back
to its original position.

12. A solid copper sphere is kept on an insulating stand. A charge given to it


gets distributed uniformly on its surface only. Which of the following
factors is/are relevant to this observation ?
(A) Copper is a conducting material.
(B) Shape of the conductor is a sphere.
(C) Like charges repel each other.
(D) Potential energy of the system is minimum in this configuration

13. A block of mass 5 kg is initially at rest on a rough horizontal surface


having coefficient of static friction s = 0.5 and coefficient of kinetic friction
k = 0.3. A gradually increasing horizontal force is applied for dragging it.
Assuming g = 10 m/s2, acceleration of the block and dragging force
acting on the block could be respectively given by,
(A) 7 m/s2, 50 N (B) 2 m/s2, 25 N
(C) 0 m/s2, 20 N (D) 3m/s2, 40 N
17
14. Two stars are seen close to each other in the sky. Star A appears brighter
than Star B. Which of the following statements satisfactorily explain the
difference in their observed brightness ? Assume both the stars to be
perfect black bodies.
(A) Both the stars are identical except for the fact that star a is closer to
us than star B.
(B) Both the stars are at same distance, but star A appears yellow,
where as star B appears orange.
(C) Both the stars are identical except for the fact that A has smaller
radius than star B.
(D) Both the stars are identical except for the fact that star A is less
massive than star B.

Section C: Analytical Questions


. (8 marks) In the following table, the first column gives the names of
various bright stars in the sky and the top row gives the names of some
zodiacal constellations. In the answers sheet, tick mark the constellation
to which they may belong. Wrong tick marks carry negative points.

. (8marks) Prof. Subramanium Chandrasekhar was first to suggest that


the white dwarf stars will have an upper limit on their mass, which is
given by

b 2
hc 1
Mlimit = K a
m
G e H
where a, e and k are dimensionless integers, with k 1 and e 2
is called mean number of nucleons per electron. mH is the mass of one
hydrogen atom. This is famously known as Chandrasekhar Mass limit
for which he won Nobel Prize in 1983. Find a and b.
18
. (8 marks) Hot solar plasma is
emitted from surface of a circular
sunspot whose diameter is 10,000
km. When the plasma reaches the
height of 16000 km above the
surface of the sun its horizontal
cross section is measured to have
diameter of 90,000 km. Assuming
that the edge of the plasma cone is parabolic, find the depth inside the
sun from which the plasma started. Assume that the viscosity and
magnetic permeability remains same inside and outside the solar
surface.
. (8 marks) Vinita Studied a star for 55 days in succession. She noted
down the temperature of the star everyday, which was varying in a nice
symmetric manner. The data of her observations is given below. Help
Vinita to find the mean temperature and theperiod of temperature
variation of this star by any suitable method. Give proper justification for
the method used.
Days Temperature Days Temperature Days Temperature
1 5472 20 5312 38 5437
2 5527 21 5264 39 5503
3 5550 22 5250 40 5543
4 5536 23 5273 41 5547
5 5488 24 5328 42 5516
6 5419 25 5400 43 5455
7 5345 26 5472 44 5381
8 5284 27 5527 45 5312
9 5253 28 5550 46 5264
10 5257 29 5536 47 5250
11 5297 30 5488 48 5273
12 5363 31 5419 49 5328
13 5437 32 5345 50 5400
14 5503 33 5284 51 5472
15 5543 34 5253 52 5527
16 5547 35 5257 53 5550
17 5516 36 5297 54 5536
18 5455 37 5363 55 5488
19 5381

(9 marks) On one fine day, Akshay was watching DTH television from
Madurai in Tamil nadu (7807E ;948N). He got a call from his IIT friend,
Sujeet, who was watching DTH television of the same complay from
Salem in tamil Nadu (7807E ;948N). Both were getting their DTH
signals from the same satellite located at 36000 km directly above a
point at the same longitude but at latitude of 1043.5N. Find the angle
difference in the antenna pointing for Akshay and Sujeet.
Sketch the graphs of following functions in the space provided on the
answersheet (Plotting on a graphsheet is not expected) :
(a) (3 marks) | x + 1 | + | x 1| (b) (3 marks) x + sin (x).
(c) (3 marks) xlog(x).
19

Indian National Astronomy Olympiad (INAO) 2011


ANSWER & SOLUTIONS

Section 1 : Multiple Choce Questions


1. (B)
Sol. Height of the orbit of polar satellite is 800 Km. After Sunset, the Sunlight
will reach satellite for a time (), where is the angular velocity of the
earth and is as shown in the diagram.
= 15 deg/ hr
Satellite
R h
= cos1 R h Sunset from observer
ground
R

6.4 10 6
R
= cos 1
6 5
6.4 10 8 10 last instant
of illumination
8
= cos1 = cos1 (0.889)
9

3
1 0

~ cos 1
(0.866) = cos 2 = 30

Thus the angle is slightly less than 300, Thus for this angle, time will be
slightly less than 2 hours

2. (B)
Sol. Let U1 be the potential energy of the object on the ground, U2 be the
potential energy at the height 2R and U be the change in the potential
energy
GM
U1 = R

GM
U2 = U = U2 = U1
3R

GM GM 2 GM
U = + U =
3R R 3R
Note : Implicit assumption is that the mass is scaled in earth mass
units. Since the assumption was not explicitly stated, the question was
deemed ambiguious and was dropped from evaluation.

3. (D)
Sol. Pole star appears stationary because it is almost along the axis of
rotation of the earth.
20
4. (A)
Sol. If the object is kept at a distance of 2f, image will also form at the distance
of 2f and that will be real and inverted. the distance between two lenses
is arranged in such a way that the image from the first lens forms at 2f of
the second lens.
Thus again a real image of this will be formed at distance 2f on the other
side of second lens and it will invert the inverted image. Thus final
image will be an erect one.

Object
Image.

5 cm f

20 cm f
40 cm 40 cm
50 cm

5. (B)
Sol. Plastic is not a conducting material so its motion will not be affected.
When ring Q will drop over the magnet, due to mutual induction eddy
currents will form which oppose the downward motion of copper ring so
it will take longer time to reach at bottom of magnet.
Since R is not complete circular ring, circuit can not be completed but
small local loops of eddy current still form in the ring so it will reach to
bottom of magnet after ring P but before ring Q.

6. (D)

Sol. Lorentzs force is given by F q VB

If initial velocity vector is restricted to only the plane perpendicular to

B ,particle would have only circular motion since particle is also
progressing along X-axis it must have parallel component along this
direction. So initial velocity should be

V = Vx i Vy j Vzk , where Vx 0 and at least one out of Vy and Vz 0

7. (D)
Sol. P should be even and 4P < 10, hence P = 2.
S is 3 or 8 and 4 P < S S = 8
Q and R are and 4 Q < 10 Q = 1 R = 7

8. (C)
Sol. From P-V diagram
Work done is = Area under the curve
Total work done by the gas = W tot = W AB + W BC
The work done would be differnce in the areas under the curve. However,
in case of work done, the negative sign is implicit in W BC.
21
9. (D)
Sol. We can simplify given circuit as follows

Rres = 6

10. (B)
Sol. Since we want it to be a prime number, last digit must be 7. Also one can
repeat same digits. Hence, there are 9 possibilities:
557, 567, 577, 667, 677, 757,767, 777
Since 5 + 6 + 7 = 18, 567 and 657 both number are divisible by 3.
Clearly 777 is divisible by 7, so we have to check remaining 6 numbers
out of which 667 is divisible by 23 and 767 is divisible by 13. Remaining
4 are prime numbers.

Section B: (4 questions 5 marks each)


11. (BD)
The Proton plate A and plate C are positively charged, hence if the
proton is displaced towards plate A, then it will get repelled and return to
its equilibrium position i.e. the centre of the assembly.
Conversely, plate B and Plate D are negatively charged, so if the proton
is displaced towards them it will get attracted.
Now if the proton is displaced diagonally, positive plates will repel it and
negative plates will attract it. Thus it will move towards the closer negative
plate. However the motion will also have a harmonic oscillator component
in direction of positive plates hence the overall motion may not be termed
as direct.
12. (BCD)
In a perfect conductor, like charges are free to take up equilibrium
positions in response to the Coulomb repulsion between them. Sphere
being symmetric, there will not be any accumulation of charges at any
point and hence there will be uniform distribution over the surface of the
sphere, this is the energetically most favorable distribution to the charge
Hence the potential energy will be minimum in this case, Since the
material of the sphere is a good conductor, all charges will only reside
on the outer surface, whether the interior is hollow or solid.
22
13. (AC)
Initially the mass is stationary therefore, the frictional force acting on it is
FS = s mg i.e 25 N.
if the external horizontal force applied is smaller than Fs, then the
acceleration produced is zero. Hence, choice 3 i.e 0,20 is correct.
Similarly, if the applied horizontal force is greater than the frictional force
then the acceleration produce can be calculated as follows :
ma = Fh Fk
For Fh = 50 N,
a = 7m/s 2
For Fh = 40 N,
a = 5m/s 2
Now at exactly Fh = 25 N, we have to consider s and not k
Om /s
14. (AB)
If both the starts are identical in mass and age, and if star A is closer to
us than star B, then its apparent brightness will be greater than star B.
Now if both the stars are at same distance and have same mass, then
star A will appear brighter if it is intrinsically bright i.e. it is hotter (blue is
hotter than yellow) than star B the size and the mass of the star does not
explicitly explain the brightness of a star. A star B having smaller radius
than star A does not specify that star B will be brighter than star A .
Similarly a more massive star need not always be brighter than a low
mass star.

Section C: Analytical Questions


. Star Name Aries Taurus Gemini Leo Virgo Scorpio
Aldebaran
Antares
Castor
Denebola
Hemal
Regulus
Sirius
Spica
Pollus

Marking scheme :
One mark for each correct answer.
* 0.5 for each wrong answer.
Sirius should be left blank. No marks for leaving it blank. Negative
marks for putting a tick mark in that row.
If row than one constellations are ticked for same star., it is counted
as wrong answer. total 8 marks.
23
. b can be found by dimensional analysis as follows ( 3 marks for
getting correct b)
Marking sxheme :
One mark for each correct answer.
b 2
hg 1
1.44 = k a m
G e H
b


J.s m / s 1
kg = 2 2
Nm
kg


kg 2

kg. m kg. m2
now, N = and J
s2 s2


m2.kg
.s m / s
s2 1

kg = kg.m 2 2
b = 3/2
m kg2
s2


kg2

Similarly a can be found by substituting the values of all the constants


given and equating it to 1.44 which is the famous Chandrasekhar
mass limit.
2
10 34 3 10 8 6.023 10 26
1.44 2 1030 = a ( )3 / 2 [1 Marks]
6.67 10 11 2 1.008


1 36 1052
= a ( 15
)3 / 2
2.22 10 4

3/2
1.44 2 10 30 2.22 1015
a
9 10 52

a 0.16 2 (2.22) 3/2 1022 1022.5

a 0.322 2.223 10
a10.9
a 3.47 a 3
24
Marking scheme :
All 4 marks for getting correct answer.
3 marks for close enough integer answers.
2 marks for getting only correct order of magnitude (i.e.0).
0.5 marks deducted if final answer is not an integer.
+ 1 mark for overlall clarity of solution.

d1 r2 1 r 2

. =
d 2 r h2
9 r h2
1 r2
=
3 r 16000
r = 80000 km (3 marks)
(2 marks) Marking scheme :
Wrong parabola (x2 = 4ay) considered. Deduct [4 Marks]
Linear terms in Y included inthe equation, grading as per merit of
justification

Day Maximum Values Temperature Days Maximum Values Temperature


3 5550 9 5253
16 5547 22 5250
28 5550 34 5253
41 5547 47 5250
53 5550

after exactly 25 days you get same maximum temperature and there is
one peak in between. so the period is 25/2 = 12.5
from the data maximum 5550
minimum 5550
mean 54000
since data contains some incomplete period, mean of all
55 readings will give incorrect answer. Since data is symmetric, mean
by merely finding peaks and taking average is reasonably correct.
25
Marking Scheme :
Period value : [2 Marks]
Period justification : [1 Marks]
Mean value [2 Marks]
Correct method for finding mean [3 Marks]

. Let x be the angle of the antenna with respect to horizon. Radius of the
earth (r) = 6400 km. satellite

l r d
A d
x 90
l
r l B
r 6400 1.85 r
90 90 Earth
d 36000 2

64 1.85 1
90 90 0.32 0.52 90
200 36
o
6
The antenna at Salem o
will be pointing 61 south of local zenith and
1
antenna
o
at Madurai 6 north of local zenith. The secular angle difference
is 31 or 20 [ 4 marks]
The total angle difference (i.e. difference in angles measured w.r.t local
horizons) would be 20 + 1 0 51 = 20 11 [ 2 marks]
20

18

16

14
.
12

10

0
10 5 5 10
26

Marking Scheme :

Criterian Graph 1 Graph 2 Graph 3


Shape 1 2 1
Values 1 2
Slope 1
Concept 1
27
INDIAN NATIONAL ASTRONOMY OLYMPIAD (INAO)-2012
INAO 2012 Date: 28th January 2012
Duration: Three Hours Maximum Marks: 100

Please Note:
Please write your roll number on top of this page in the space provided.
Before starting, please ensure that you have received a copy of the
question paper containing total 3 pages (6 sides).
In Section A, there are 10 multiple choice questions with 4 alternatives
out of which only 1 is correct. You get 3 marks for each correct answer
and -1 mark for each wrong answer.
In Section B, there are 4 multiple choice questions with 4 alternatives
each, out of which any number of alternatives may be correct. You get
5 marks for each correct answer. No marks are deducted for any wrong
answers. You will get credit for the question if and only if you mark all
correct choices and no wrong choices. There is no partial credit.
For both these sections, you have to indicate the answers on the page
2 of the answersheet by putting a in the appropriate box against the
relevant question number, like this:

Marking a cross () means affirmative response (selecting the particular


choice).
Do not use ticks or any other signs to mark the correct answers.
In Section C, there are 5 analytical questions totalling 50 marks.
Blank spaces are provided in the question paper for the rough work. No
rough work should be done on the answer-sheet.
No computational aides like calculators, log tables, slide rule etc. are
allowed.
The answer-sheet must be returned to the invigilator. You can take this
question booklet back with you.
28
Section 1:Multiple Choice Questions
Part A: (10 Q 3 marks each)
1. If the square of your age in seconds gives the age of the universe in
seconds, then
(A) you havent started to walk yet
(B) you are in primary school
(C) you are a young adult
(D) you were born before british left india

2. If f(x + y) = f(x) f(y) and f(2) = 5, then the value of f(2) is


(A) 5 (B) 1 (C) 0.25 (D) 0.2

3. A container with uniform rectangular cross-section and weight M kg,


falls from a cargo ship in to the sea and is floating with x part of its height
under water (x is a fraction less than 1). Two persons from a nearby
boat, each weighing m kg board on the container resulting it to go down
such that the top surface of the container levels exactly with the level.
M
Find the ratio in terms of x.
m
1 x x 1 x 2x
(A) (B) (C) (D)
2x 1 x x 1 x
4. An electron moving uniformly in space is neither deflected nor accelerated
over a long distance. Which of the following statements may describe
the local conditions ?
(A) Both electric and magnitic fields are necessarily zero simultaneously
(B) The electric field is necessarily zero.
(C) The magnetic field is necessarily zero
(D) Neither of them are necessarily

5. Pradip once observed image of the sum on the platform of CST station
(Mumbais main railway station). He realised that the sunrays forming
the image are entering through a tiny triangular shaped window in the
high ceiling. He measured the diameter of the image to be 0.175 m.
Find the height of the selling.
(A) 12.5 m (B) 18.75 m (C) 22.5 m (D) 37.5 m
6. The most energy efficient direction of projection of rockets from the earth
surface is
(A) Eastwards (B) Westwards
(C) Northwards (D) Vertically upwards
7. A car travels the first 20 km eastwards with 30 km/hr, next 20 km
northwards with 40 km/hr and then 20 km westwards with 50 km/hr. The
average velocity of the car for the journey is
10 600 400
(A) 10 km/hr (B) km/hr (C) km/hr (D) km/hr
9 47 47
29
8. The sum is found to be settings exactly at 6.00 pm on a given day. If the
Earths atmosphere was only half dense as it is then the sunset would
have occurred
(A) Slightly later than 6.00 pm
(B) Slightly earlier than 6.00 pm
(C) Exactly at 6.00 pm
(D) It depends on the latitude of the place

9. A knock-out tennis tournament, begins with a total of 193 players. In


each round, If the number of player is odd, then one player gets direct
bye to the next round. No player can get byes in two consecutive rounds.
How many matches must be played in the tournament before the winner
is decided ?
(A) 386 (B) 192 (C) 169 (D) 97

10. Four metal rods all of identical diamensions and made of same material
are welded together at a single point. The configuration is such that any
two rods are oriented at 120. The far ends of three of the rods are
maintained at 90 temperature and that of the fourth rod is maintained at
30 temperature. The temperature of the junction point is
(A) 45 (B) 60 (C) 75 (D) None of these

Section : B (2 Questions 5 Marks)


11. Consider the following statements :
(i) A central eclipse is the one where central point of lunar disk exactly
passes over the central point of solar disk.
(ii) An eclipse is called a total Eclipse if it is seen as total from at least
some point on the earth.
(iii) An eclipse is called a partical Eclipse if it is not seen as total /
annular from any point on the earth.
Now choose the incorrect statements from below :
(A) All central eclipses are total
(B) All total eclipses are central
(C) All particle eclipses are non-central
(D) All non-central eclipses are partial

12. Consider a pendulum with inextensible string with a lightweight magnetic


bob. Underneath this arrangement iron dust is spread out. Now the bob
of the pendulum starts swinging. Which of the following might be a
susequent observations ?
(A) The maximum height reached by the bob will start increasing
(B) The tension in the string will start increasing
(C) The period of oscillation of the bob will start decreasing
(D) The angular momentum of the bob remains constant
30
Section C : Analytical Questions
() (10 marks) In the night sky, the constellations depict several shapes of
animate objects (species like birds, animals, etc) or man made
instruments. List 20 such figures / instruments. The list should contain
at least 4 instruments. Write both the object and the corresponding
constellation. Exact name of the constellation is not necessary. The
constellations must be from International Astronomical Union's standard
list of 88 constellations (i.e., Indian / Chinese / Mayan constellations not
accepted).

() From the graph below, find :


(a) (7 marks) The cubic polynomial describing the curve.
(Note : f(7) 200]
(b) (5 marks) If this curve describes orbit of a comet with the Sun at
point (1,0) and the comet crosses the x-axis exactly two months apart,
find the approximate position of the comet 1 month since first crossing
of the x-axis.

. (a) (6 marks) Assume that human civilisation has carved out all the
mantle and the central core of the earth, and started living on the
inside surface of the earth's crust which is only about 7 km in
thickness. As a consequence, people are finding that their weight
has alterted drastically. To feel normal weight again, one scientist
suggested an idea involving tampering one of the motions of the
earth. Can you describe this idea qualitatively and quantitatively ?
31
(b) (8 marks) An observer is sitting in a stationary spacecraft outside
earth's orbit such that at the closest point, distance between the
observer and the earth is 1 AU. Sketch a rough plot (exact functional
form / shape not expected) showing how the orbital angular velocity
of the earth varies with time (in an year), as seen by this observer.
Mark every point of maximum or minimum clearly and write its
coordinates. Similarly, mark every point with zero angular velocity
and write its coordinates. Assume the orbit of earth around the Sun
to be circular.

In the 1957 science fiction novel "The black cloud" by Sir Fred Hoyle, an
interstellar dark cloud is discovered approaching the solar system. We
will try to retrace the steps explained in the novel to calculate speed and
direction of the cloud. Two photographs given here show a small region
inside the constellation of Orion. The photographs are taken exactly
2 months apart. The grid in the image has the size of 1 1. By
spectroscopy, it was realised that the emission line of neutral hydrogen
(Rest wavelength 21.1000 cm) emitted by this cloud was showing up at
21.0789 cm wavelength.

(a) (4 marks) In how months (after the second image) the cloud will
entirely cover the belt of Orion (seen on the left) ?
(b) (1 mark) Is the cloud headed directly for the solar system ? Why?
(c) (2 marks) If yes, then in how many months (after the second image)
the cloud will arrive at the earth ? If no, what will be the closest
separation between the cloud and the earth ? Assume the cloud to
have uniform velocity throughout the journey.
(d) (5 marks) If this spherically symmetric cloud is placed at the exact
centre of the solar system, which planets will be engulfed by the
cloud ?
Note: The expression of relativistic Doppler effect is given by observed = emitted

cv
cv
32

(12 marks) Let us assume a situation where an asteroid passes close


to the earth. Some times the shortest distance. In such cases, the asteroid
will cross the lunar orbit at two points. A particular asteroid makes a
closeearth pass such that at its closest proximity to the earth, its
distance from the earth is 256000 km. For simplicity, let us assume that
the gravitational force of the earth acts on this asteroid only for a short
duration of 1000 seconds, when the asteroid is exactly at the closest
proximity point. Let us assume that the asteroid follows straight line
paths with uniform velocities before and after this gravitational interaction
and the velocity of the asteroid before the interaction was 6.25 km/s.
Find out how much would be the deviation measured (in km) along the
lunar orbit. Assume the Moon to be on the other side of the Earth in its
orbit during the asteroid's transit, hence not having any impact on the
motion. Take the orbit of the Moon around the Earth to be circular.
33

INDIAN NATIONAL ASTRONOMY OLYMPIAD (INAO)-2012


ANSWER KEY WITH SOLUTION
1. (D)
Sol. Age of the Universe is about 14 billion years. If your age in years is Y.
then,
(Y 365.25 86400)2 = 14 109 365.25 86400

14 10 9
Y2 =
365.25 86400

14
103 500
28

Y 500 22 Years
Thus, your age is early 20s.

2. (D)
Sol. f(x + y) = f(x) f(y)
Put y = 0 f(x) = f(x) f(0) f(x) (1 f(0)) = 0
f(x) 0, f(0) = 1
Put x = 2 and y = 2 f(0) = f(2) f(2)
1 = 5 f(2) f(2) = 1/5 = 0.2
Alternate :
f(4) = f(2) f(2) = 25 and f(2) = f(4 2) = f(4) f(2) = 25f (2)

f (2) 5
Thus : f(2) = = = 0.2
25 25
Solution 2 :
f(2) = f(2 + 0) = f(2) f(0) implying f(0) = 1
Thus, f(0) = f(2 2) = f(2) f(2)

f (0 ) 1
Giving, f(2) = = = 0.2
f ( 2) 5
3. (A)
Sol. If V is volume of the container.
M = V

M
& M + 2m = V M + 2m =

M 2
2m = M(1 ) =
m (1 )
34
4. (D)
Sol. The Lorentz force equation is given by,

F q(E v B )

We require F to be zero. The electric and magnetic fields can be so
adjusted that their net effect cancels out, and the electron continues its
path.
5. (B)
Sol. h = height of the ceiling ; Dse = 1.5 1011 m ; = diameter of image;
d = 14 108 = diameter of sun.

h 1.5 1011
.175 h 18.75m
14 10 8
6. (D)
Sol. The earth rotates from west to east. If the rocket is projected in a eastward
direction, the earths motion acts as a boost for the projection.
7. (C)
Sol. Total displacement 20 km

20 20 20
Total time
30 40 50

20 600
avg. vol. = = km/hr
20 20 20 47

30 40 50
8. (A)
Sol. We see the sunset a few minutes after geometric sunset as the solar
disk remains visible even after going below horizon due to refraction. If
the atmosphere is rarer, the sunrays will get refracted less. Thus, this
additional period of visibility of solar disk is reduced. Hence, we will
see sunset sloghtly earlier.
9. (B)
Sol. Round No. No. of players No. of matches
1 193 96
2 97 48
3 49 24
4 25 12
5 13 6
6 7 3
7 4 2
8 2 1
Total number of matches = 192
35
Original Sol.
The only way a player can go out of the tournament is by losing. Winner
is a single player; thus, 193 1 = 192 players must lose. Implying 192
matches.

10. (D)
Sol. For equilibrium condiction heat flow in and out of junction should be
same. Let k present constant representing heat content, which may
depend on dimensions and material of the rods.
k(T 30) = 3k (90 T)
4T = 300
T = 75

Section : B (2 Questions 5 Marks)


11. (BD)
Sol. First One is incorrect as some central eclipses can be annular only.
Second one is incorrect as one can get a total eclipse even when the
two disks are slightly off-centre w.r.t. to each other.

12. (BC)
Sol. As the oscillation starts the bob will slowly pick up iron dust and its
mass will increase. As the velocity of the bob at the hightest point is zero,
by conservation of energy, height of the highest point must decrease. As
the mass increases, the tension in the string will increase too. Since the
string is inextensible the time period will remain the same. But angular
momentum, L = will change, as changes. ( is moment of inertia).

Section C : Analytical Questions


() Animals(easy): bull(Taurus), ram(Aries), eagle(Aquila), bear(UrsaMajor/
UrsaMinor), lion(Leo), crab(Cancer), scorpion(Scorpio), dog(CanisMajor/
CanisMinor), fishes(Pisces), swan(Cygnus), dolphin(Delphinus),
snake(Serpens), crow(Corvus), man(Orion/Bootesetc.)
Animals(difficult): fly(Musca), hare(Lepus), Wolf(Lupus), fox(Vulpec-ula),
peacock(Pavo), dove(Columba), Emu(camelopardalis), smallhorse
(Equ-uleus), swordfish (Dorado), crane(Grus), lizard(Lacerta),
Chamaeleon, hornbill (Tucana
Animals(mythical): watergoat(Capricornus), centaur(Centaurus),
dragon(Draco), wingedhorse(Pegasus), amythicalsnake(Hydra),
aseamonster(Ce-tus), abird(Apus), Phoenix
instruments: Telescopium, scales(Libra), Microscopium, Sextans,
directionalcompass(Pyxis), Table(Mensa), cross(Crux), Crown (Corona
Australis/Corona Borealis), harp (Lyra), arrow (Sagitta), shield (Scutum),
easel (Pictor), airpump (Antila), compasses (Circinus)
Each correct creature / instrument: 0.5 marks. Maximum 16 creatures
allowed.
36
() (a)From the graph, we note that f(4) = 0, f(8) = 0, f(0) = 800
and f(1) = 840. Using these to find the cubic polynomial, (2.5 Marks)
f(x) = ax3 + bx2 + cx + d
0 = a(4)3 + b(4)2 + c(4) + d = 64a + 16b 4c + d
0 = 83a + 82b + 8c + d = 512a + 64b + 8c + d
800 =d
840 =a+b+c+d
40 =a+b+c
& 0 = 576a + 48b + 12c
c = 48a + 4b + = 4(12a + b)
& 0 = 64a + 8b + c + 100
0 = 24b + 3c + 900
300 = 8b + c
Solving, we get,
f(x) = x3 29x2 + 68x + 800 (1)
(b) We have to divide the area enclosed by the curve and the x-axis in 2
equal halfs as divided by the line from point (1,0), to get the position for
the comet after one month.
Non-calculus solution: A rough counting of squares shows that the
correct position would be approximately between x = 2 and x = 2.5.
Calculus based solution: If the x-coordinate of the comet after one month
was x0,
8 x0 1
f ( x )dx 2 f ( x )dx ( x 0 1)f ( x 0 ) (2)
4 4 2
Solving, we get,

x 04 32x 30 88
+ 29x 02 868 x0 + 64x =0 (3)
2 3 3
X0 = 2 yeilds slightly positive value and X0 = 3 yeilds large negative value.
Thus, the root the the equation lies very close to 2 but is higher than 2.
Note: The approximate solution turns out to be X0 = 2.2071.
Marking Principle: By either way, student should state (with proper
justification) that the x-coordinate is close to 2 but higher than 2.

. (a) For a person standing on the inside surface of the hollow shell,
there is no effect of gravity due to the shell itself (shell theorem).
He / she would feel the weight purely due to the centrifugal force due to
the rotation of earth. (2 marks) Thus, to get the normal weight back, the
earth must start rotating faster. (0.5 mark)
Shell thickness should be considered as negligible as compared to the
radius of the earth. (0.5 mark)
g = R2
37

2 R 6.4 x10 6
= = 2
T g 10
2 x 800 T 5000 sec
Thus, the earths rotation period should be decreased to about 84
minutes.
(2 marks)
This calculation is valid only for the equator and apparent weight at other
latitudes would be different.
B

60 Earth
Sun
Observer
(b) A
C

The exact expression for the function above is

() = 0 sin() cos( )
sin()

R sin()
Where tan =
k R(1 cos())
In this, R is the orbital radius of the earth (1 AU), k is the shortest separation
between the observer and the earth orbit (also 1 AU), is the observer
- sun earth angle, is the earth - observer - sun angle. The coordinates
for points of interest would be (0, 0), (60,0), (180, 0/3) and (300,0),
where x-axis represents . This can be recalibrated in terms of time
with 30 interval corresponding to one month period.
38
Thighn of as well as start of the year point are matters of convention.
Hence if this graph is flipped on x-axis or is shifted along the x-axis, the
answer is equally valid.
Marking scheme: 1.5 mark for each the four coordinates. 2 marks for the
overall shape of the curve and proper markings of axes etc.

The size of the cloud is about 1 cm in the first image and 2 cm in the
second image. Now, Mintaka and Alnitak are almost equidistant from
the clouds centre (about 4.6 cm) so they would be get covered at almost
same time. If the physical diameter of cloud is x and its distace from us
is d,
x = d = 2(dt2) = 9.2(dt3)
d = 2t2
& 9.2t3 = 7.2d + 2t2 = 16.4t2
16.4
t3 = 2 months = 3.565 months
9 .2

i.e. In just over a month from the second image (about 34 days), the belt
of Orion will be covered by the cloud.
(b) The cloud is headed directly for the solar system as the position of
its centre has not shifted from the first image to the second image.
(c) D = 2 t2 = t4
Thus, the cloud will arrive at the solar system in exactly 2 months after
the second image is taken
(d) Velocity of the cloud is given by
2
c observed 21.1 (1 0.001)
0.998
c emitted 21.1
A per the grid, about 1.95 cm correspond to 1 separation in the sky.
Thus 0.5 in the forst image.
x = d = 2 t 2

2 3 10 5 2 30 86400 24 8.64
x= 0 .5
1.5 1011 180 360


1.728
3
x 108 A.U.
Thus the cloud radius would about 0.9 A.U. Meaning the cloud will engulf
Mecury and Venus.
39

Path after C
deviation s
B D
Path if
undeviated
C
Earth Moon
d
A
(shortest E
distance) R B F

Approach


at
GMt 6.67 10 11 6 10 24 10 3
uj i = 6.25 103 j + i
d2 (2.56 10 8 )2

6250 i +601 i
As the first term on the R.H.S. is much larger as compared to the second
term, the angle of deviation of the asteroid would be very samll. In the
figure, E denotes centre of the Earth. A closest proximity position of
asteroid. BC is are measures along the lunar orbit. AB is denoted by r
and both EB and EC would equal to R. As angle of deviation is very
small, Arc length BC can be approximated as length BD. Let angle BAC
be and angle AEB be . Thus, angle FBC and hence approximately
angle FBD will also be 90 - . Angle BFD can be approximated to 90.
at
tan () = 10 3 tan () 10 3
u
T
sin () =
R
BF r tan( )
BD = cos(90 ) = = R tan () 3084 10810-3
sin()
s 384km
Note : If BC is approximated to BF, only 3 marks would be awarded.
secondly AB = r R. That approach does not any points.
40

Indian National Astronomy Olympiad (INAO) 2013


INAO 2013 Date: 2nd February 2013
Duration: Three Hours Maximum Marks : 100

1. (10 marks) Astrophysicist Freeman Dyson proposed around 50 years


back that an advanced civilisation would make optimal use of the energy
of the parent star by constructing a full shell around the parent star with
radius equal to the orbital radius of their planet, trapping all the radiation
inside. The civilisation can live on the surface of this shell. We would like
to construct a Dyson sphere with radius equal to the orbital radius of
the Earth. Let us assume that we have access to all the material within
the solar system (except the Sun itself, which we would like to retain as
the energy source) and we have necessary technology to solidify the
material available to us. What will be rough thickness of this shell?

2. (10 marks) Following table contains names of a few constellations and


names of some stars. Fill in the blanks. For missing star names, you
can name any star from that constellation.

STAR CONSTELLATION
Polaris

Rigil Kentaurus

Orion

Cygnus

Taurus
Sirius

Vega

Regulus
Virgo

Scorpio

3. (10 marks) An Indian festival called Kojagiri is always celebrated on a


Full Moon night in Autumn. As a part of festivities, many people keep a
bowl of milk on the ground or a terrace under the Moon light for a few
minutes, before drinking it. On this years Kojagiri night, Sheetal had
placed roughly 100 ml milk in a dish of size 25cm diameter for about 10
minutes exactly when the moon was overhead. Assume that density of
milk is similar to the water, its specific heat capacity is 0.9 times that of
water and heat loss from the milk to surroundings is negligible. Assume
that full Moon reflects about 14% of the sunlight incident on its surface.
What will be the change of temperature caused by the light received
from the moon?
41
4. (7 marks) Let A and B be two objects in the solar system orbiting each
other and A is much heavier than B. Theoretically, what is the maximum
possible revolution period of B ?

5. (8 marks) Prove that 2013 20124 + 1 is a composite number.

6. (10 marks) Ayush was doing an experiment with a lens in his Physics
lab. He kept a reference object at several positions and noted
corresponding image sizes and positions. The same are shown in the
diagram below. Unfortunately, he forgot to label the object and image
positions and to mention details about the lens, because he thought
them to be too trivial. Help his science teacher to make sense of his
drawing by finding type of lens (convex / concave), its position and its
focal length. In the diagram, O stands for Object and I for image. All
lengths and heights are to scale.
I
I
o o I
I

o o o

7. (5 marks) In a competition for the most useless inventions, Sharad


presented a novel technique which makes it possible to fold an ordinary
paper 50 times on itself, without any gaps between layers of folds.
Calculate thickness of the folded paper after it has been folded 50 times.
Note that the paper is folded further and further without opening the
previous folds i.e. it is folded in half and then the folded paper is folded
on the middle to reduce the area to a quarter and so on.

8. (10 marks) Alankar was jogging downhill and he noted distance traveled
by him at various intervals. Plot appropriate graph to find his acceleration
and initial velocity. In the table, time (t) is in minutes and distance (d) is
in meters. Both the quantities are measured from the start of the journey.

t d t d t d t d t d
2 21 6 117 10 285 14 525 18 837
4 62 8 192 12 396 16 672 20 1020

9. (10 marks) Sandesh fabricated a magic sphere of radius R, which is


hollow on the inside and has perfectly reflecting inner surface. This
sphere had a small hole in it. Sandesh sent a ray of light radially through
this hole. It hit a plane mirror, kept at an angle of 45? with the incident ray,
at some point beyond the centre of the sphere, but before reaching the
opposite end. After undergoing one more reflection at the inner surface
of the sphere, the ray came out from the hole. Find the distance from the
centre of the sphere to the point where it struck the plane mirror.
42

10. (20 marks) Answer following questions in 3 to 4 lines each.

(a) The core of earth is like a perfectly conducting fluid sphere. If this
core suddenly shrinks to half of its present radius, then what will
happen to its magnetic field ? Why?

(b) A satellite is revolving around the Earth in a polar orbit with 90 minute
period. On 23rd September, Prasad saw it exactly overhead at the
time of sunset. Within next twenty four hours how many sunrises
will be seen by the satellite? Why?

(c) Anand has taken a high zoom photograph of a sun-spot. The


photograph does not include any part of the disk outside sun-spot.
What will be the predominant colour seen in the photograph? Why?

(d) What will happen to the Moon if the Earth vanishes suddenly? Why?

(e) On a full Moon day, with respect to an observer on the Earth, does
the Moon move faster (angle covered per hour) during the day time
or the during night time? Why?
43
Indian National Astronomy Olympiad (INAO) 2013
ANSWER KEY WITH SOLUTION
1. Total mass available to us for constructing the Dyson Sphere would
primarily consist of mass of all planets and satellites. As an
approximation, we will say Jupiter and Saturn are of similar size. Saturn
is known to have very low density so its mass will not exceed half Jupiter
mass. Further, radius of Uranus and Neptune is less than half of Jupiter
radius. Thus, they will be at least 10 times lighter than Jupiter. Similarly,
Earth and Venus are of similar size and Mars is about half of that. There
are a handful of objects with radii about a quarter of that of Earth or even
smaller (Mercury, Moon, Ganymede, Callisto, Titan etc.). All these are
rocky bodies and we can take typical density to be s = 5 gm/cc. Combined
mass of all other combined bodies will not be greater than mass of
Mars. As we want shell to be solid like rock, we will assume similar
density for the shell too. So if the shell has thickness ,
J M M M
4D2s 2MJ 2 10 2M 2 10 6 100

2MJ 0.2MJ 2M 0.2M 0.06M


4D 2s

2.2(MJ M ) 2.2(2 1027 6 1024 ) 2006 10


3
3mt
4(1.5 1011 )2 5 103 22
4 2.25 10 5 10 200
Thus, the thickness will be approximately of the order of a few meters.
As is evident from the numbers, mass estimates are just rough indicators
of total mass.

2. STAR CONSTELLATION
Polaris Ursa Minor
Rigil Kentaurus Centaurus

Betelgeuse Orion

Deneb Cygnus

Aldebaran Taurus

Sirius Canis Major


Vega Lyra

Regulus Leo

Spica Virgo

Antares Scorpio
44
3. Moon receives light energy from the Sun which is partly reflected back
towards the Earth.
L
Solar Energy per unit area incident on Moon =
4 D 2
L
Total Solar Energy incident on Moon = R m2
4D 2
L
Total Solar Energy reflected by Moon = R m2
4 D 2

m L R 2
Reflected Energy received at Earth per unit area = 4 D 2 2d2 Er

Let us assume that this entire energy is used to heat the milk without
any looses.
tErA = msT
tr 2 L R m2
T = Vs 4D 2 2d2
m

600 0.125 2 3.826 10 26 (1.7 10 6 )2 0.14


=
10 103 0.9 4.18 103 8 (1.5 1011 )2 (3.84 10 8 )2
4

1
T 1.1 104
7000
Thus, temperature will rise by only about 104 Celsius.

4. to have longest possible period, the orbital radius should also be longest.
If A was any planet, orbital radius of B cannot be very high as far from the
planet B will come under influence of the Sun. So for longest possible
radii, we take A as the Sun and try to place B as far as possible.
The other limiting factor will be the influence of other stars. As B still part
of the solar system, Sun exerts more influence on it than any other star.
Star closest to us is the Alpha-Centauri triplet. It has two stars similar to
the Sun and Proxima Centauri, which is much smaller than the Sun.
Thus, we can approximate the system as about 2M system about
4.3 light years away.
rd
1
Thus, at about of the distance, Alpha-Certanuri system will start
3
dominating on B. This means that maximum possible radius is about
4.3
1.4 light years. Thus, longest period will be,
3
We can utilise the fact that orbital radius of the Earth is 1 A.U. and orbital
period is 1 year.
45

4 2 3
T2 = r
GM
T12 R13

T22 R 32

R13 (1.4 63240 )3


T1 = T2 3 = 1
R2 13

(90000)3 = 3003 = 2.7 107 years


Thus, approximate period would be 27 million years.
It is acceptable if Alpha-Centauri system is assumed to have mass of
just a M.

5. Solution :
X = 2013 20124 + 1
= (2012 + 1) 20124 + 1
= (a + 1) a4 + 1 where a = 2012
= a5 + a4 + 1
= a5 + a4 + a3 a3 + 1
= a3(a2 + a + 1) (a3 1)
= a3(a2 + a + 1) (a 1)(a2 + a + 1)
= (a2 + a + 1)(a3 a + 1)
= (20122 + 2013)(20123 2011)

6. The type of lens is convex lens. In the picture, Object-Image pairs have
been given same subscript. O5 is placed at exact focus so it has no
corresponding image. Focal length of lens can be measured from the
image.
I4
I3
I2 o5 o4 I1

o1 o3 o2

7. A 100 page notebook is roughly 1 cm thick. Thus thickness of a single


paper is 102 cm or 104 mt. If you fold the paper 50 times, its thickness
will become,
= 250 104 (1)
= (210)5 104 = (1024)5 104 (2)
1015 104 (3)
1011 mt (4)
Thus, the thickness will be of the order of the Sun-Earth Distance.
46

1 2
8. d = ut + at
2
Thus, if one plots a graph of distance versus time, it will be a parabolic
curve.
d a
To linearise the graph, we change the equation as u t
t 2
d a
Now if we plot, versus t, it will give a linear graph with as slope and
t 2
u as y-intercept.

t d/t t d/t t d/t t d/t t d/t

2 10.5 6 19.5 10 28.5 14 37.5 18 46.5

4 15 8 24 12 33 16 42 20 51

f(x) = 2.25x + 6
60

50

40
t/
d 30

20

10

0
2 4 6 8 10 12 14 16 18 20
t

Thus, u = 6 m/s and a = 4.5 m/s2.


9. In the figure below, D is the point of second reflection. will be normal
to the surface at D. D
BDO = ADO = DAO = x AO = OD
OBD = 90 A O B
AOD = 180 ( ADO + DAO) = 180 2x
but AOD = OBD + BDO
180 - 2x = 90 + x x = 30
R
(OB) = (OD) sin 30 =
2
47

1
10. (a) Magnetic flux must be conserved. Thus, B . As the radius
R2
shrinks to half, the magnetic field will increase four-fold.
(b) As the satellite makes angle of 90? with the Sun, its orbit is exactly
along the line separating day and night regions on the Earths
surface. Hence, its one side will continuously face the Sun and
hence, there will be NO sunrises seen by the satellite.
(c) The sunspots have temperature of around 4500K. By Wiens law,
this corresponds to roughly red colour. Thus, the sunspots will
appear red.
(d) Moon will retain all its velocity at the instance of disappearance of
the Earth. Thus, it will have significant non- radial velocity around
the Sun and it will continue to orbit the Sun in near identical path.
(e) We note that Moon revolves in the same direction as the rotation of
the Earth
i.e. from West to East. On the full Moon day, Moon is on the night
side of the Earth. Thus, with respect to an observer on the Earth, the
tangential velocities of the Moon and the Earth during the night will
be in the same direction and hence relative velocity is slower. During
the day tangential velocities are in opposite directions hence the
relative velocity is faster. Thus, Moon will be moving faster during
the day.

Notes for Junior Group


Question 4 : Take body A to be the Sun. Keplers third law states that the
square of orbital period will be proportional to the cube of orbital radius.
Question 5 : a3 1 = (a 1)(a2 + a + 1).
Question 8: Modify appropriate equation of motion such that all points
will lie along a straight line.
Question 10 (a) : If the conducting sphere suddenly changes the size,
the field lines will stay trapped on the surface.
Question 10 (c): At any given temperature, the peak of thermal radiation

Wien' s constant
will be at wavelength max =
T
48

INDIAN NATIONAL ASTRONOMY OLYMPIAD (INAO) - 2014


INAO 2014 Date: 1st February 2014
Duration: Three Hours Maximum Marks: 100

1. (12 marks) Read the following passage and point out scientific
inaccuracies. Give very brief argument for each mistake you point out.
Arrival of a spacecraft on the surface of the Moon is called a Moon landing.
This includes both manned and unmanned (robotic) missions. In order
to get to the Moon, a spacecraft must first cross the point, beyond which
the Earth's gravitational force is zero. The only practical way of
accomplishing this currently is with a rocket. At every instant, the rocket
has to produce thrust to propel itself to a velocity equal to critical velocity
at that height. Jet engines can be used to propel the spacecraft to the
Moon, but rockets provide greater advantage in terms of power required
for given mass of the spacecraft.
Unlike the Earth, the Moon does not have thick atmosphere to absorb
most of the solar radiation and the magnetic field of the moon is too
weak to deflect the UV rays. Thus, astronauts traveling to the Moon are
exposed to harmful electromagnetic radiation. Spacesuits for astronauts
are specially designed by keeping this in mind.
On arrival near the Moon, the spacecraft is captured by the lunar gravity
in an orbit around the Moon. As the Moon is much smaller than the Earth,
typically these orbits are low altitude orbits (close the lunar surface) as
compared to the however, if one needs it to land on the Moon, it has to
fire its engines to change course.
Landing at the Moon can be of two types. A hard landing is equivalent to
crash landing on the Moon. A soft landing is a controlled landing in
which priority is to maintain all equipment ad astronauts (if any) inside
the spacecraft safe. Needless to say, all Apollo missions had a soft
landing on the Moon. This was achieved by firing reversing rockets very
close the lunar surface to slow down the spacecraft. Charring of the
lunar soil, due to burning of these rockets, is one of the permanent
impressions left by humankind on the Moon.

2. (6 marks) A comet is in elliptical orbit around the Sun with period T and
semimajor axis a. A second comet around the Sun has same period T, but
a different orbit with same eccentricity e. The two orbits are not necessarily
in the same plane. Further, we also know that when the first comet is
closest to the Sun in its orbit (at perihelion), the other comet is farthest from
the Sum in its orbit (at aphelion). Given this configuration, what is the
minimum possible distance between the two comets, when one of the
comets is at its perihelion ? Justify your answer by brief arguments
sketches.
49
3. On a flight between the cities of Oslo (61N, 8E) and Helsinki (60.2N,
25E), Mayank saw the Sun just at the horizon on the west. After 20
minute, when he checked again, he saw the Sun was no 2 above the
horizon.
(a) (2 Marks) The flight was traveling from which city to which city ?
(b) (8 Marks) What was the speed of the aircraft with respect to the
ground (in Km/hr)
(c) (4 Marks) How long will this aircraft take to traverse the distance
between these two cities ? Assume uniform speed during the entire
flight.

4. The skymap below corresponds to sky above Nagpur (21N, 79E at


09.00 am on 1st February 2014. If you are not used to using sky maps,
it is important to note that sky map is usually seen lying down on the
ground (feet to the South), facing the sky with map in your hand. Thus,
East is on the left of the map and West is on the ring. Answer the following
questions :
50
(a) (1 marks) Mark Polaris with letter P.
(b) (2 marks) Circumpolar stars for a given place are the stars, which
will never go below the horizon. Draw boundary of this region and
mark it by the letter C.
(c) (2 marks) The celestial equator is just a projection of the Earth's
equator in the sky. It will be locus of points which are equidistant
from the north and the south pole. Draw the equator on the map
approximately and mark it with Q.
(d) (2 marks) The ecliptic is the imaginary yearly path of the Sun in the
sky. Mark this approximately on the map and mark it with E.
(e) (2 marks) Mark approximate position of the Sun on the map as S.
(f) (2 marks) Yesterday was a new moon day. Mark the current position
of the moon on the map as M
(g) (2 marks) Which star was very close the Zenith at 06:00 am today ?
Mark it on the map as N.
(h) (2 marks) Draw a line across sky showing horizon line as at
07:00 am today as H.

5. (12 Marks) Gliese-876 is a star in the constellation of Aquarius which


has four confirmed planets going around it. We know following facts
about this system :
The four planets have names b, c, d and e.
The four planets have masses (in Earth masses) of 7M, 15M,
225M, and 720M. (not necessarily in the same order)
The orbital radii of the four planets (in A.U.) are 1.93, 30, 61, 124.
(not necessarily in the same order)
The orbital eccentricities of the planets are 0.03, 0.06, 0.21, 0.26
(not necessarily in the same order)
Planet e is farthest from the star.
The heaviest planet has the lowest eccentricity.
The lightest planet is also the closest to the star.
The two giant planets are neither the closest nor the farthest from
the star.
The planet with a mass of 7M has eccentricity of 0.21 and is closer
to the star than planet c.
Eccentricity of planet d is more than planet e, but less than c.
Planet c is closer to the star as compared to planet b.
Use the information above to correctly identify mass, orbital radius
and orbital eccentricity of each of the planet. For your help, you may use
the grid given in the answer sheet. The grid has six sub-grids of 4 4
size. You can tick mark in the boxes corresponding to matching pairs
and cross mark the boxes where there is no match. Note that this grid is
only for your help. It is our choice if you with to use this grid or some other
method. Final answer must be written in the second table below the
grid. Only the values written in the table will be considered for giving
marks.
51
6. (a) (9 marks) A thin convex lens, of focal length of f, is placed in y-z
plane. The principal axis of the lens is along the x-axis. A luminous
square sheet is placed in the x-y plane with its sides parallel to
x and y axes. The centre of the sheet is exactly at a distance of 2f
from the center of the lens and length of each side of the sheet is f.
The sheet is slightly rotated about the y-axis to make the entire
sheet visible from the other side of lens. Sketch the shape of the
image of the sheet and mark all relevant lengths in terms of f.
(b) (6 marks) A pin P is placed in front of a concave mirror such that its
mid-point lies on the principal axis at a distance d from the pole of
the mirror. An observer on the principal axis at a distance D, where
D >> d, finds the pin and its real image to overlap. When the observer
moves slightly to the left of the principal axis, the image is viewed to
the right of the pin. From this observation, what can be concluded
about value of d? Justify your answer.

7. (8 marks) Chandrayaan-1, the lunar mission launched by India in 2008,


had all gold wiring. A particular instrument on board the mission, was
operating at the same temperature as the space surrounding the
chandrayaan. Instrument specifications demanded that no wire in the
instrument should offer a resistance more than 7mW. All the instruments
on Chandrayaan were fabricated in ISRO labs at room temperature. If
the radius of all wires in the instrument was 0.0 mm (as measured in
ISRO lab), what should be the maximum allowable length (as measured
in ISRO lab) of any wire? Physical properties of gold :
Resistivity (at room temperature), = 2.214 108m
Temperature coefficient of resistivity, = 0.0032 K1
Linear expansion coefficient, =1.5 105K1
Density, r = 2 104 kgm3
Molar heat capacity, Cp = 25 J mol1 K1
Molecular weight, W 197 g

8. (18 marks) We are in the year 2020 and the spacecraft Vogager-1 us at
a distance of around 200 A . U. from the Sun. It is still continuously
sending information about its speed with respect to inertial frame of the
surrounding interstellar medium every 10 days. Following are the
readings reported by the Voyager over course of 100 days.
52

Time (days) 0 10 20 30 40 50

Velocity (m/s) 1000 1061.2 1124.4 1189 1255.4 1323.8

Time (days) 60 70 80 90 100

Velocity (m/s) 1394.4 1467.4 1543.1 1621.8 1703.8

Swapnil claimed that this indicates existence of an unknown body directly


ahead in the path of Voyager. Those of you who agree with him, find
mass and distance of this unknown body from Voyager. Those who
disagree with him should give alternate explanation for the data observed
and justify their arguments.
Note :
For purpose of any calculations, you may ignore the influence of the
Sun on Voyager. At the end of calculations, you should give reason
why this is a valid assumption. Two points are reserved for this
justification.
As an approximation, assume the acceleration of the Voyager to be
constant during each 10 day period between the readings.
You may not that 8.52 72.
For calculation, you may assume one day approximately contains
90000 second.
53

INDIAN NATIONAL ASTRONOMY OLYMPIAD (INAO)-2014


ANSWER KEY WITH SOLUTION

SECTIONS-I : MULTIPLE CHOICE QUESTIONS


Section : A (10 Questions 3 Marks)
1. Earth's derivational force goes to zero only at infinity. Close the Moon, the
force is small but not zero.
Velocity needed is bigger than the critical velocity.
For jet engines, the oxygen is taken from the air. It is not stored on-
board. So they cannot be use the take spacecraft to the Moon.
UV rays are electromagnetic waves and are not affected by the
magnetic field.
Low altitude orbits are not stabler, as imperfections on the lunar
surface result in change in gravitational force from point to point.
The lunar soil won't get charred due to firing of rockets as there is
no oxygen on the lunar surface.
One mark for each correct identification of the mistake. One mark
each for corresponding reasoning.
If a blatantly working argument is resulting in incorrect identification
of an inaccuracy, one mark will be subtracted.
No negative marking for subtle mistakes in reasoning.

2. By visualising the configuration correctly, it will be realised that to achieve


minimum possible separation, the two orbits should lie in the same
plane with their semi-major axes along the same line, such that
sum - perihelion - 1-Thus, the minimum separation will be [3 Marks]
d = daphelion, 1 dperihelion, 2
= a(1 + e) a(1 e)
= 2ae [3 Marks]
3. The Sun rising from the west can be observed only if we are traveling in
the direction opposite to the Earth's rotation. Thus, the flight was from
Helsinki to Oslo. [2 Marks]
In 20 minutes, the Earth would rotate by 5 eastwards. Mayank saw the
Sun rising by 2 in that time. This means, the plane moves (5 + 2)
westwards in 20 minutes. [2 Marks]
We can approximate latitudes of both places to = 60
= 5 + 2 = 7
7
= = rad s1
20 60 180 1200 180
7
v = r = R cos = 6.4 106 cos 60
1200 180
7 1 28 22 8800 18
= 64 326 m/s 1170 km/hr
1.2 18 2 0.27 7 27 5
54
[4 Marks]
Difference between longitudes of the two cities is 17. Thus,
17 20
t= 49 min.
7
The flight will take about 49 minutes. [4 Marks]
Note : The solution above is a simplification as q is not estimated correctly.
When Maynak will see the Sun 2 above horizon, it would actually mean
that at 60 latitude, the sun has rolled back by 4 along its apparent path.
Thus, solution using = 9 is more correct. However, students are not
expected to know this at INO level and hence solution given about is
accepted for full credit.
4. Notes :
Pole star should be exact.
Circumpolar region should be a circle around P with radius equal
to Polaries-Northern horizon distance.
Equator should be a smooth curve passing through East and West
points and below zenith (centre) in the middle
Escliptic should be a smooth curve roughly passing through the
zodiacal signs (i.e., mostly below equator) and cutting equator in Virgo.
The Sun should be in Capricornus.
The Moon should be in Aquarius.
Any star reasonably close the Archturus is accepted as answer for N.
Horizon at 7 : 00 am should not cut current northern horizon, roughtly
pass close to the Sun and cut bottom edge on the west of south.
55

5. Mass Eccentricity Orbital Radius

b 720 0.03 61

c 225 0.26 30

d 7 0.21 1.93

e 15 0.06 124

One mark per value

6. (a) The image will look as follows :


f

Image
object f
3f 2f 5 f f 3 2f 5
f f f
3 2 2
2

Correct drawing of object : 1 Mark


Correct extremities of image on the axis : 3 Marks
Top and bottom edges NOT curved : 2 Marks
Left and right edges curved on correct side : 3 Marks
(b) As you move you observing position laterally, the closer objects
appear to move faster than the farther objects.
Thus, when you move your eye to the left, the one closer out of the object
and the image will appear to shift to the right of other. [3 Marks]
It means that image distance is more than object distance. [1 Mark]
Hence, the object is f and 2f. [2 Marks]

7. Mass of the wire is,


M = r12 1 r22 2 r12 1 r22 2


Resistance of wire is given by, R = =
A r 2
2 2
2 = 1(1 T) ; 2 = 1(1 T) ; R2 = [2 Marks]
r22
2
1(1 T ) 2 1(1 T ) 2 1(1 T ) 1(1 T )2
= = =
r2 1r12 1 21
11
2

1(1 T 2T 2 T 2 ) 1
= [2 Marks]
r12
56
Typical room temperature can be assumed to be 30 Celcius.
Temperature in space is typically 3 Kelvin. Thus, temperature difference
will be about 300 Kelvin. [1 Marks]
Now, as 2 term is too small. Further, as is 100 times smaller than
beta, it can be ignored too. [1 Marks]

1(1 T ) 1
R2
r12

r12R 2 (10 4 )2 7 10 3 22
2 (1 T )
1 2.214 10 8 (1 0.0032 300 ) 7

10 2 1
(1 0.96 ) 4 = 25 cm [2 marks]

Thus, the wire can be at max 25 cm long.

8. We note that Voyager's velocity is changing. As we are ignoring influence


of the Sun, this can only be because of presence of some unknown
object close by. By noting difference between first two readings and last
two readings, we realise that the change is not the same, i.e. acceleration
is changing. We try to estimate average acceleration and velocities for
each period by,
v u
a=
t
s 1
vav = u at
t 2
In the table below, time is in days, velocities in m/sec and acceleration in
m m/s2. Average velocity of the Voyager is given by vav in m/s and last
column is cumulative sum of vav. [8 Marks]
t v vu a vav vav

0 1000

10 1061.5 61.5 68.3 1031 1031

20 1124.4 62.9 69.9 1093 2124

30 1189 64.6 71.8 1157 3280

40 1255.4 66.4 73.8 1222 4503

50 1323.8 68.4 76.0 1290 5792

60 1394.4 70.6 78.4 1359 7151

70 1467.4 73.0 81.1 1431 8582

80 1543.1 75.7 84.1 1505 10087

90 1621.8 78.7 87.4 1582 11670

100 1703.8 82.0 91.2 1663 13333


57
Now note that,
s = vavt
s = vav 10 90000

GM
a=
(r0 s)2
a1(s1)2 = a2(r0 (s)2)2

a1 (r0 s)1) = 81.1 (r0 8582 90000)

r0 r0
8.5 8
3.3 9 8
8 .6
9 10 9 10
r0 9 108 (8.6 18 3.3 17) 9 108 98.7
8.9 1010 m 0.6 AU [5 Marks]
Now we use the is distance to find mass of the object.

a(r0 s)2
M=
G

98.3 10 6 (8.9 1010 1031 9 105 )2 3



20 10 11

68.3 (8.8)2 10 24 3
68.3 77.4 1.5 1024
2
5.3 103 1.6 1024 8 1027 kg 4MJ [3 Marks]
Now the object is about 300 times lighter than the sun and about 400
times closer to the Voyager as compared to the Sun
2
Fobj Mobj rsun 4002
2

FSun robj MSun 300
Thus, gravitational force by the Sun on the Voyager will be about 500
times smaller and hence can be safely ignored.
58

Indian National Astronomy Olympiad -2015


st
Date : 31 January 2015
Duration : Three Hours Maximum Marks: 100

Please Note :
Please write your roll number on top of this page in the space provided.
Before starting, please ensure that you have received a copy of the
question paper containing total 3 pages (5 sides).
There are total 9 questions. Maximum marks are indicated in front of
each question.
For all questions, the process involved in arriving at the solution is more
important than the answer itself. Valid assumptions / approximations
are perfectly acceptable. Please write your method clearly, explicitly
stating all reasoning.
Blank spaces are provided in the question paper for the rough work. No
rough work should be done on the answer-sheet.
No computational aides like calculators, log tables, slide rule etc. are
allowed.
The answer-sheet must be returned to the invigilator. You can take
this question booklet back with you.
Please be advised that tentative dates for the next stage are as follows
:
th th
Orientation Camp (Junior) : 29 April to 7 May 2015
th th
Orientation Camp (Senior) : 2 May to 7 May 2015
th th
Selection Camp (Jr. + Sr.) : 26 May to 5 June 2015
Participation in both parts (Orientation and Selection) is mandatory for
all participants.
Useful Physical Constants
Mass of the Earth ME 6 1024 kg
Radius of the Earth RE 6.4 106 m
Mass of Jupiter MJ 2.0 1027 kg
8
Speed of Light c 3.0 10 m/s
Astronomical Unit 1 A.U 1.5 1011 m
26
Solar Luminosity L 3.8 10 W
3
G 20 10 m kg s
11 1 2
Gravitational constant
3
Avogadro constant Na 6.023 1023 mol1
23 1
Charge of an electron e 1.6 10 mol
7 2
Permeability of free space 0 4 10 NA
17 8 2 4
Stephan's constant s= 10 Wm K
3
59

1. In science fiction novel The Evitable Conflict, Isaac Asimove


suggested that, in a futuristic world, all food items may be replaced
with a nutritious powder made of wheat flour. What size of world
population can be sustained with this food staple ? You may assume
that the average annual yield of wheat to be about 3.6 tonne per
hectare (present day actual figure) and the geography of the Earth to
remain similar to the present world . [10 Marks]
2. Match the following (each entry in column A may map with zero or
one or more entries in column B but each entry in column B has
exactly one matching entry in column A)
Column-A Column-B
Satellite Whirlpool
Galaxy Sirius
Star Ceres
Star cluster (group) Pegasus
Dwarf Planet Pallas
Asteroid Puppis
Constellation Pluto
Planet Phobos
Pleiades
pollux
3. Two identical satellites A and B are launched in equatorial circular
orbits of period 1.8 hours. Satellite A rotates in the sense of rotation
of the Earth while satellite B rotates in the opposite sense. The orbits
are separated slightly to avoid collision. On a particular day, at 12
noon, both the satellites were seen exactly overhead by an observer
on the Earth's equator. What is the minimum duration of time after
which both these satellites will again be seen exactly overhead from
the same place ?
4. For each of the following statements, state if the statement is true or
false and given a single line justification for your answer in each
case.
1. All Stars except pole star rise in the east and set in the west.
2. The Earth's axis is changing its direction slowly because its
magnetic pole is not properly aligned with the geographic pole.
3. Cosmic rays are not part of electromagnetic spectrum.
4. Some sunspots can be bigger than the Earth.
5. Jupiter can fit in the space between the Earth's surface and the
Moon's orbit.
6. That Earth does not fall into the Sun is a direct consequence of
the Newton's third law of motion.
7. The time taken by Mars Orbiter Mission (Mangalyaan) from the
moment it left the Earth's orbit till it was captured by the gravity of
Mars, is well determined by the Kepler's laws.
8. On a new moon day, the Earth's shadow covers the moon fully.
9. The stars which appear brighter are also closer the Sun.
10. Pole star is visible from all the locations from the Earths
60

5. A big hall is 15 meter long, 10 meter wide and 5 meter high. Two ants
are seated inside this hall at one of the upper corners. On the edge,
diagonally opposite to the ants, there is a sugar cube at a height k
meters above the ground (see diagram). One of the ants decides to
go down the vertical edge, across the floor diagonal, and then up the
opposite vertical edge to reach the sugar cube. The second ant tries
to take shortest route along the walls without touching either the floor
or the ceiling. Both the ants move with the same uniform speed and
you can assume that no time is spent in changing the direction at any
point. Find condition on k such that the first ant reaches the sugar
cube first.

15 Sugar

10
Ants
5

6. A ray of light enters an assembly of plane mirrors ( from left) as


shown in the figure. below and undergoes reflection at all the four
mirrors. After the last reflection, the ray travels in vertical direction
and enters in the detecting instrument. in the figure, the dotted lines
are normals for respective mirrors and dashed lines are exactly
vertical or horizontal. The third mirror is exactly horizontal. It is known
that angle is 50. Angle f is unknown. Find angle p.
61
2
7. A spaceship sends a proton beam with cross-sectional area of 100m
at a speed of 0.01 c in the plane of the Earth's magnetic equator. The
5
closest distance of the beam to the centre of the earth is 1.28 10
km. The magnetic field at the midpoint of the distance separating the
proton beam and the center of the earth is zero.
(a) Find the number of protons emitted by the spaceship per
second.
(b) As seen by the alien controlling the proton beam from the
spaceship, would the Earth be on the left side or right side?
(c) If mass of the spaceship is 1667 tonne, how much will be its
recoil velocity due to this proton beam.
Note : The strength of the Earth's magnetic field as measured on the Earth's
5
surface at magnetic equator is about 5 10 Tesla

8. India plans to launch N communication satellites, all of which are to


be in a geostationary orbit around the equator. All satellites are
arranged equi-spaced in the same orbit. Find all the possible values
of N such that each satellites can see all the other satellites of the
series at any instant of time.

9. The given graph is a scattery-plot of all exoplanets from the Open


Exoplanet catalogue, including those discovered by the Kepler
mission. This Kepler mission is a special space based instrument,
which has been continuously observing same small part of the sky
from the start of its operation to measure any small changes in the
brightness of the individual stars in that particular direction. The
scales on the graph are as follows :
Top - log of orbital period of the planet in years.
Right - log of mass of the planet as a multiple of mass of the Earth
Bottom - log of orbital radius of the planet
Left - log of mass of the planet as a multiple of mass of jupiter
For example, on the left side, 0 would correspond to log10
(m/MJ) = 0 i.e., m = MJ. As a way to simplify the problem, let us
assume that all the parent stars of these exoplanets have exactly
same mass as that of the Sun.
(a) Complete the scale for the log10P (years) and log10m (ME) axes
by providing values on the respectively axes.

(b) Mercury's distance from the Sun is about 40% that of the Earth
and its mass is about 20 times smaller. Neptune's orbital radius
is 30 times larger than the Earth's orbital radius and Neptune is
17 times more massive than the Earth. Mark the approximate
positions of Mercury and Neptune on the graph.
We further assume that all the parent stars emit same total
energy per second (called Luminosity) as the Sun and all
exoplanet orbits are circular.
62

(c) If a planet can sustain liquid water on its surface, it is said to be


in the habitable zone around the parent star. Mark approximate
edges of the habitable zone on the graph. State clearly the
assumptions made in the process.
On way of detecting exoplanets is called the transit method
(green dots in the graph). If a planet crosses (transits) in front of
its parent star's disk as seen from the earth, then the observed
visual brightness of the star drops by a small amount because
the planet blocks some of the light. Most of the exoplanets
detected by this method were discovered by the Kepler mission,
which will complete its sixth year of operation in a few months
time. To confirm a detection, it is necessary that at least three
transits of the given exoplanet should have been observed by
the kepler mission.

(d) Assume that the average density of the exoplanet is almost the
same as that of the host star. It is known that the kepler mission
can detect about 0.01% variation in the light coming from the
star. Using this information, draw two straight lines indicating
limit for the transit method by kepler spacecraft. Give brief
justification of your answer.

Another way is called the radial velocity method (blue dues).


Here, we note that the host star and the exoplanet are both
orbiting around the common centre of mass (C.M) and hence the
host star shows a periodic variation in its velocity towards or
away from the observer (us). As the planet is not emitting any
light, we just measure the velocity of the host star using the
Doppler effect. This orbital velocity of the host star around the
common centre of mass can help us determine mass of the
exoplanet.
63

(e) Suppose the smallest change in radial velocity possible to detect


by a certain spectrograph is 8 m/s. If we draw a straight line on
the graph showing this limit, what would be the slope of this line?
Draw this line on the graph. Looking at the exoplanets plotted on
the graph, can we conclude that there exist some other
instruments that could detect smaller changes in the radial
velocity ? Justify
Note : Although these two methods account for most of the exoplanets
discovered, few of the exoplanets (also shown in this plot) are discovered by
other methods likes direct imaging, microlensing and timing analysis.
64

Indian National Astronomy Olympiad -2015


st
Date : 31 January 2015
Duration : Three Hours Maximum Marks: 100

HINTS & SOLUTIONS


2 14 2
1. Surface Area of the Earth = 4 r = 5.2 10 m . Only 30% of it is land.
14 2
Thus, total land mass is about 1.5 10 m . By rough estimate,
ignoring mountains, deserts, arid regions, forests, towns / cities etc.,
only 10% of the land is cultivable. Thus, cultivable land is only
13 2
1.5 10 m i.e. 1.5 billion hectare.
If all this land is used to cultivate wheat then, annual total wheat
9
production will be 1.5 10 3600 = 5.4 trillion kg.
Now each person on average needs 2000-3000 calories per day, which
may be obtained by less than half kg of wheat flour. One can estimate
daily intake in different ways but the number obtained would be roughly
similar. e.g. One can argue that a typical human eats about 250 gm of
food per meal and you need 3 meals per day. Thus total is 750 gm.
However, water content in all food items is more than half. Thus, actual
weight of food is only half of that. Or one can argue each roti needs
about 25 gm of flour and if you are not eating anything else, then one
may end up eating 15-20 rotis per day.
Thus, each person would need about 400 gm wheat flour per day. i.e.
about 145 kg per year.
12
i.e. sustainable population would be 5.4 10 / 145 = 40 billion people.
Note: As with all estimation questions, the arguments presented and
validity of assumptions is more important than the exact numbers.

2. Satellite Phobos Galaxy - Whirlpool


Star - Sirius, Pollux Cluster - Pleiades
Dwarf Planet - Pluto, Ceres Asteroid - Pallas
Constellation - Pegasus, Puppis Planet

3. As the earths rotation period is 24 hours, we have to find apparent time


period of these two satellites with respect to an observer on the Earth.
A = 0 E
1 1 1 1 1 24
= = ; TA = = 6 hours
TA T0 TE 4.8 24 (5 1)
24
B = 0 + E ; TB = = 4 hours
(5 1)
L.C.M. of the two apparent periods is 12 hours. Thus, the two satellites
will be seen overhead together after 12 hours.
65

4. 1. False. As an example, several stars around pole star would


never go below horizon.
2. False. Although the Earths axis is indeed precessing, this is not
related to magnetic field of the Earth.
3. True. Cosmic rays is a term used for particle showers.
4. True. The Suns radius is 100 times bigger than the Earth and
many times we see big sunspots on the solar disk. A sunspot
which is barely of the size of the Earth will not be visible through
small telescope.
5. True. Jupiter is just 10-15 times bigger than the Earth whereas
lunar orbit is 60 times bigger.
6. False. Action and reaction forces act on different bodies and
hence reaction force cannot be seen as the one balancing the
gravitational force.
7. True. The path taken by MOM after leaving the sphere of
influence of the Earth was essentially part of an elliptic orbit
around the Sun.
8. True. From the time it left the Earths orbit till the time it was
captured by Martian gravity, its trajectory was part of an effective
orbit around the Sun.
9. False. On a new moon day, the moon is between the Sun and the
Earth. So the Earths shadow cannot fall on the Moon.

5. The path length for the first ant = h + 2 w 2 k


2
The path length for the second ant = w (h k)2
Our condition is,
2
h+ 2 w 2 k < w (h k)2

5+ 152 102 + k < (15 10) (5 k)2


(5 + k) + 325 < 625 (5 k)2
2
squaring, (5 + k) + 325 + 2 325 (5 + k) <625 + (5 k)2
2
25 + 10k + k + 325 + 10 13 (5 + k) < 625 + 25 10k +
2
k
10 13 (5 + k) + 20 K < 300.
5 13 + 2 13 k < 30

5 (6 13)
k<
(2 13)
approximately, k < 2.
66

6. Let angle of incidences at the four mirrors be i1,i2,i3 and i4 respectively.


At mirror 1,
(90 ) + p = 90 i1
i1 = p
For ray from mirror 1 to mirror 2,
90 + + i1 + 90 i2 + 180 = 360
+ i1 = i2 +
i2 = 2 p
For ray from mirror 2 to mirror 3,
90 i2 + f + i3 = 180
i2 + i3 = 90
(2 p ) + i3 = 90
i3 = 90 + 2 p 2

For ray from mirror 3 to mirror 4,


i3 = 2i4
2i4 = 90 + 2 2 p
At mirror 4,
90 + i4 + = 180
i4 + = 90
2i4 + 2 = 180
90 + 2 2 p + 2 = 180
p = 2 90
= 2 50 90
p = 10.

7. (a) Let 2r0 the shortest distance between the proton beam and the
Earth. Let BE be strength of the magnetic field at the Earth's
surface (with effective pole strength ME). Let BOE be the
magnetic field at r0 due to earth's magnetic field and BOP is the
magnetic field due to p. As close to the Earth's surface, the
Earth's magnetic field acts like a dipole,
0ME
BE =
4R3e
0ME 3
= RE BE
4
M R3 B
BOE = 0 E3 = E 3 E
4 r0 r0
r0 = 10 RE
BOE = 0.001 BE
0 p
BOP =
2 r0
BOE BOP
67

0 P
0.001 BE =
2 r0
0.001 5 10 5 6.4 107
P = 0.001BEr0 =
2 107
2 0
4
7
P = 1.6 10 A
Charge on a proton is same as that on an electron
P = eN
7
N = 1.6 10
1.6 1019
26
= 10 protons per second.
(b) The field lines from the Earth's magnetic field go from (geographic)
southern hemisphere. To cancel this field, by virtue of right hand
rule, the spaceship should be placed such that for an observer
on the ship, the Earth would be seen on the right side.
(c) As protons are same as Hydrogen ions, one can say NA protons
would weigh about 1 g. Thus,
Mshipvship = Mbeamvbeam
3 1 103 1 1026 3 10 6
vship = 1 10 N 0.01 c =
NAMship 6 1023 1660 103
vshp = 0.3 m/s.

8. For geostationary orbits


20
2 10 11 6 1024 (86400)2
a = GME2TE = 3
3

4 40
2 21
= 8.64 10
2 7
a 8.64 10 .
(R + h) 42000 km
First of all, if the number of satellite are even then for every satellite,
there will always be another satellite at diametrically opposite position.
Clearly any satellite cannot see the diametrically opposite satellite. So n
cannot be even.
If we draw tangents to the Earths surface from any satellite (as shown
in the figure), it becomes clear that minimum angle between the two
consecutive satellites as seen from the Earths would be BCD.
68

BCD = 2 BAD
= 2 2 ABC
1 R 4 6400
= 4 sin
R h 42000
4
rad
7
As a circle has 2 radians, N can take following values :
N {3, 5, 7, 9, 11}
Note : Exact calculations show that N should be less than 11. Thus, 11 will
be taken as marginal case. solutions with or without 11 will get full
credit.

9. (a) The orbital radius of the Earth is 1 AU and its orbital period is 1
year. Thus, for the Earth.
log10 a (AU) = log10p (year) = 0
Using this one can put marks on the top scale. Note that the
2 3
markings are closer than bottom scale as p a .
mJ
For right side scale, we not that = 300. Thus, the mark
mE
approximately at the same level as 0 on the left scale corresponds
to ratio of 300. Using this we make marks on the right scale.
(b) Use the orbital radius and mass ratio values to put these planets
at appropriate place.
(c) Let us assume the planet is in thermal equilibrium. Thus, amount
of heat absorbed would be same as the amount heat emitted.
L r 2 2 4
= 4r sT
4a2
1 L
a= 2
.
4T
We notice that this equation is independent of planet parameters. Thus,
the boundaries will be vertical lines. At inner boundary, the equilibrium
temperature should be 273 K and at outer boundary it should be 373 K.
1 4 1026 1 1017
aout = =
4 (270)2 22 17
10 8 4 7.3 10 4 1.5 1011
7 3
4
17
100
= 1.1 A.U.
2 4 7.3 1.5
69

100
ain = 0.6 A.U.
2 4 13.7 1.5
Only approximate estimation is expected here.
(d) As the observer (us) is very far from the star, the physical
separation between planet and the star is negligible and the drop
in intensity of star light during a planet transit depends only on the
ratio of cross-sectional area. Thus, one should draw a horizontal
line.
1
2
A pl Rpl Vpl 2
= =
A st R st Vst
2
Mpl 3
0.0001 =
Mst
3/2 Mpl
0.0001 =
Mst
6
Mpl = 1 10 Mst = 0.001 MJ.
Thus, one should draw horizontal line where log10 m(MJ) = 3. Further,
as mentioned in the begining, Kepler has been in operation for nearly 6
years and it required at least 3 transits to flag a detection. Thus, typical
orbital period for exoplanets discovered by Kepler should be 2 years or
smaller. Draw corresponding vertical line.

(e) As the orbit is circular, vst = astw and moments about the centre
of mass are balanced.
mstast = mplapl
mplapl mplapl 2 mpl2 apl2 42
vst = astw = w= ; v 2st 2
mst mst T mst T mst
mpl2 apl2 G G mpl
2

by Kepler's third law, v 2st =


mst a3pl mst apl
Thus, for any given velocity detection limit,
G
apl = mpl2
mst v 2st
GME2
apl (in A.U.) = mpl2 (in ME2 )
(1 A.U.)mst v st2
1 GME2
[log10 apl ] 1 log10

[log10 mpl] =
2
2 2 (1 A.U. m v )
st st
70


1 20 10 11 (6 10 24 )2
= [log10 apl ] 1 log10
2 2
3 1.5 1011 2 1030 18
2


1 1
= [log10 apl ] log10 (10 3 )
2 2
1
= [log10 apl ] + 1.5.
2
This is equation of a straight line in the form y = mx + c. Thus, on the
given plot, we have to draw a straight line with slope 0.5. Further, points
(2, 0.5) and (3, 3) are on the line. Once we draw the line, we realise
that a lot of blue points are below this line. Thus, present detection limit
should be lower than this value.

Resonance Eduventures Ltd.


CORPORATE OFFICE : CG Tower, A-46 & 52, IPIA, Near City Mall, Jhalawar Road, Kota (Raj.) - 324005
Ph.No. : +91-744-3012222, 6635555 | Toll Free : 1800 200 2244 | 1800 102 6262 | 1800 258 5555
Reg. Office : J-2, Jawahar Nagar, Main Road, Kota (Raj.)-324005 | Ph. No.: +91-744-3192222 | FAX No. : +91-022-39167222
Website : www.resonance.ac.in | E-mail : contact@resonance.ac.in | CIN: U80302RJ2007PLC024029

Você também pode gostar